ChaseDream

标题: 【每日逻辑练习第二季【2-2】 [打印本页]

作者: UlysessHope    时间: 2011-12-10 10:48
标题: 【每日逻辑练习第二季【2-2】
下午要去外地,所以中午先发上练习吧~待我考完回归
大家加油~
【精练】
3. Modern navigation systems, which are found in most
of today’s commercial aircraft, are made with
low-power circuitry, which is more susceptible to
interference than the vacuum-tube circuitry found in
older planes. During landing, navigation systems
receive radio signals from the airport to guide the
plane to the runway. Recently, one plane with
low-power circuitry veered off course during landing,
its dials dimming, when a passenger turned on a
laptop computer. Clearly, modern aircraft navigation
systems are being put at risk by the electronic
devices that passengers carry on board, such as
cassette players and laptop computers.
Which one of the following, if true, LEAST
strengthens the argument above?
(A) After the laptop computer was turned off, the
plane regained course and its navigation
instruments and dials returned to normal.
(B) When in use all electronic devices emit
electromagnetic radiation, which is known to
interfere with circuitry.
(C) No problems with navigational equipment or
instrument dials have been reported on
flights with no passenger-owned electronic
devices on board.
(D) Significant electromagnetic radiation from
portable electronic devices can travel up to
eight meters, and some passenger seats on
modern aircraft are located within four meters
of the navigation systems.
(E) Planes were first equipped with low-power
circuitry at about the same time portable
electronic devices became popular.

【逻辑链】
13.(25752-!-item-!-188;#058&002668)
An unusually severe winter occurred in Europe after the continent was blanketed by a blue haze resulting from the eruption of the Laki Volcano in the European republic of Iceland in the summer of 1984.  Thus, it is evident that major eruptions cause the atmosphere to become cooler than it would be otherwise.
Which of the following statements, if true, most seriously weakens the argument above?
(A) The cooling effect triggered by volcanic eruptions in 1985 was counteracted by an unusual warming of Pacific waters.
(B) There is a strong statistical link between volcanic eruptions and the severity of the rainy season in India.
(C) A few months after El Chichn's large eruption in April 1982, air temperatures throughout the region remained higher than expected, given the long-term weather trends.
(D) The climatic effects of major volcanic eruptions can temporarily mask the general warming trend resulting from an excess of carbon dioxide in the atmosphere.
(E) Three months after an early springtime eruption in South America during the late 19th century, sea surface temperatures near the coast began to fall.

14.(25938-!-item-!-188;#058&002908)
Most household appliances use electricity only when in use.  Many microwave ovens, however, have built-in clocks and so use some electricity even when they are not in use.  The clocks each consume about 45 kilowatt-hours per year.  Therefore, households whose microwave oven has no built-in clock use 45 kilowatt-hours per year less, on average, than do comparable households whose microwave oven is otherwise similar but has a built-in clock.
Which of the following is an assumption on which the argument depends?
(A) Households that do not have a microwave oven use less energy per year, on average, than do households that have a microwave oven.
(B) Microwave ovens with a built-in clock do not generally cost more to buy than microwave ovens without a built-in clock.
(C) All households that have a microwave oven also have either a gas oven or a conventional electric oven.
(D) Households whose microwave oven does not have a built-in clock are no more likely to have a separate electric clock plugged in than households whose microwave oven has one.
(E) There are more households that have a microwave oven with a built-in clock than there are households that have a microwave oven without a built-in clock.

15.(25986-!-item-!-188;#058&002914)
Some airlines allegedly reduce fares on certain routes to a level at which they lose money, in order to drive competitors off those routes.  However, this method of eliminating competition cannot be profitable in the long run.  Once an airline successfully implements this method, any attempt to recoup the earlier losses by charging high fares on that route for an extended period would only provide competitors with a better opportunity to undercut the airline's fares.
Which of the following, if true, most seriously weakens the argument?
(A) In some countries it is not illegal for a company to drive away competitors by selling a product below cost.
(B) Airline executives generally believe that a company that once underpriced its fares to drive away competitors is very likely to do so again if new competitors emerge.
(C) As part of promotions designed to attract new customers, airlines sometimes reduce their ticket prices to below an economically sustainable level.
(D) On deciding to stop serving particular routes, most airlines shift resources to other routes rather than reduce the size of their operations.
(E) When airlines dramatically reduce their fares on a particular route, the total number of air passengers on that route increases greatly.

16.(26678-!-item-!-188;#058&003277)
When an airplane is taken out of service for maintenance, it is often repainted as well, and during the repainting no other maintenance work can be done on the plane.  In order to reduce maintenance time, airline officials are considering using a new nontoxic plastic film instead of paint.  The film takes just as long to apply as paint does, but many other maintenance tasks can be carried out at the same time.
Which of the following, if true, is further evidence that using the film will help the airline officials achieve their goal?
(A) Unlike paint, the film gives a milky tone to certain colors.
(B) At the end of its useful life, the film can be removed much more quickly than paint can.
(C) The film can be applied only by technicians who have received special training.
(D) The metal exteriors of airplanes have to be protected from high temperatures and caustic chemicals such as exhaust gases.
(E) Even at speeds considerably higher than the normal speed of a passenger jet, the film remains securely attached.

作者: UlysessHope    时间: 2011-12-10 10:49
逻辑链答案:CDBB
精练解析:
The conclusion of the argument is based on the causal assumption that
electronic devices cause a disturbance in low-power circuitry, creating an
obvious danger:
ED = electronic devices
I = interference with low-power circuitry
C                   E
ED    →         I
The question stem is a StrengthenX (remember, Least works like Except in
question stems) and thus the four incorrect answers will each strengthen the
argument. As you attack the answer choices, look for the five causal
strengthening answer types discussed earlier.
Answer choice (A): This answer choice strengthens the argument by showing
that when the cause is absent, the effect does not occur (Type C). Once the
laptop was turned off, the cause disappeared, and according to the author’s
beliefs, the effect should then disappear as well.
Answer choice (B): This answer strengthens the argument by showing that the
data used to make the conclusion is accurate (Type E). By stating that all
electronic devices emit radiation, the answer choice closes a hole in the
argument.
Answer choice (C): This answer choice strengthens the argument by showing
that when the cause is absent, the effect does not occur (Type C).
Answer choice (D): This answer strengthens the argument by showing that the
data used to make the conclusion is accurate (Type E). By showing that
radiation can travel far enough to reach the cockpit, the cause is confirmed as
possible.
Answer choice (E): This is the correct answer. The fact that the circuitry and
electronic devices became popular at the same time does not offer any
supporting evidence to the contention that the electronic devices cause the
interference with the low power circuitry. This answer has no effect on the
argument and is therefore correct.
作者: fox0923    时间: 2011-12-10 10:58
我来占位~~
祝hope考试成功!!

-----------------------------------------------------------------------------
精练-------------------42s--------------------least support
B: The modern navigation system is used for the airplane.
P: The low power circuitry dials dimming when a passenger turns on his laptop on the airplane.
C: Therefore, the navigation system are put on the risk if the passengers carry electric equipment on the airplane.
Prephrase: The screening system always checks the electricity brought by the passengers before boarding.


Analysis:
(A) After the laptop computer was turned off, the
plane regained course and its navigation
instruments and dials returned to normal.------------------------support
(B) When in use all electronic devices emit
electromagnetic radiation, which is known to
interfere with circuitry.-----------------------------------------------support relation of reason-effect
(C) No problems with navigational equipment or
instrument dials have been reported on
flights with no passenger-owned electronic
devices on board.-----------------------------------------------------conclusion:
electronic devices推 navigation system's failure; 这里是no electronic devices推no navigation system's failure.---〉support
(D) Significant electromagnetic radiation from
portable electronic devices can travel up to
eight meters, and some passenger seats on
modern aircraft are located within four meters
of the navigation systems.-------------------------------------------4 meters< 8 meters, so the navigation system will be affected by the radiation
(E) Planes were first equipped with low-power
circuitry at about the same time portable
electronic devices became popular.--------------------------------planes became popular??? certainly it's irrelevant!

作者: qiuhua01234567    时间: 2011-12-10 12:22
zhan
作者: xeyyxzty    时间: 2011-12-10 13:41
1.Clearly, Which one of the following, if true, LEAST strengthens the argument above?B:Modern navigation systems, which are found in most of today’s commercial aircraft, are made with low-power circuitry, which is more susceptible to interference than the vacuum-tube circuitry found in older planes. During landing, navigation systems receive radio signals from the airport to guide the
plane to the runway. P:Recently, one plane with low-power circuitry veered off course during landing, its dials dimming, when a passenger turned on a laptop computer.
C:modern aircraft navigation systems are being put at risk by the electronic devices that passengers carry on board, such as cassette players and laptop computers.
is there a link between the laptop and the dials
(A) After the laptop computer was turned off, the plane regained course and its navigation instruments and dials returned to normal.
--support
(B) When in use all electronic devices emit electromagnetic radiation, which is known to interfere with circuitry.
--support
(C) No problems with navigational equipment or instrument dials have been reported on flights with no passenger-owned electronic devices on board.
--support
(D) Significant electromagnetic radiation from portable electronic devices can travel up to eight meters, and some passenger seats on modern aircraft are located within four meters of the navigation systems.
--support
(E) Planes were first equipped with low-power circuitry at about the same time portable electronic devices became popular.
--irrelevant--right
2.P:An unusually severe winter occurred in Europe after the continent was blanketed by a blue haze resulting from the eruption of the Laki Volcano in the European republic of Iceland in the summer of 1984.  
C: it is evident that major eruptions cause the atmosphere to become cooler than it would be otherwise.
(A) The cooling effect triggered by volcanic eruptions in 1985 was counteracted by an unusual warming of Pacific waters.
--irrelevant
(B) There is a strong statistical link between volcanic eruptions and the severity of the rainy season in India.
--irrelevant
(C) A few months after El Chichn's large eruption in April 1982, air temperatures throughout the region remained higher than expected, given the long-term weather trends.
--right
(D) The climatic effects of major volcanic eruptions can temporarily mask the general warming trend resulting from an excess of carbon dioxide in the atmosphere.
--irrelevant
(E) Three months after an early springtime eruption in South America during the late 19th century, sea surface temperatures near the coast began to fall.
--irrelevant
3.
B:Most household appliances use electricity only when in use.  Many microwave ovens, however, have built-in clocks and so use some electricity even when they are not in use.  
P:The clocks each consume about 45 kilowatt-hours per year.  
C:households whose microwave oven has no built-in clock use 45 kilowatt-hours per year less, on average, than do comparable households whose microwave oven is otherwise similar but has a built-in clock.
the average time using microwave oven is the same
(A) Households that do not have a microwave oven use less energy per year, on average, than do households that have a microwave oven.
--irrelevant
(B) Microwave ovens with a built-in clock do not generally cost more to buy than microwave ovens without a built-in clock.
--irrelevant
(C) All households that have a microwave oven also have either a gas oven or a conventional electric oven.
--irrelevant
(D) Households whose microwave oven does not have a built-in clock are no more likely to have a separate electric clock plugged in than households whose microwave oven has one.
--right
(E) There are more households that have a microwave oven with a built-in clock than there are households that have a microwave oven without a built-in clock.
--irrelevant
4. B:Some airlines allegedly reduce fares on certain routes to a level at which they lose money, in order to drive competitors off those routes.
P: Once an airline successfully implements this method, any attempt to recoup the earlier losses by charging high fares on that route for an extended period would only provide competitors with a better opportunity to undercut the airline's fares.
C:this method of eliminating competition cannot be profitable in the long run.
???
(A) In some countries it is not illegal for a company to drive away competitors by selling a product below cost.
--irrelevant
(B) Airline executives generally believe that a company that once underpriced its fares to drive away competitors is very likely to do so again if new competitors emerge.
--irrelevant--right
(C) As part of promotions designed to attract new customers, airlines sometimes reduce their ticket prices to below an economically sustainable level.
--irrelevant
(D) On deciding to stop serving particular routes, most airlines shift resources to other routes rather than reduce the size of their operations.
--right
(E) When airlines dramatically reduce their fares on a particular route, the total number of air passengers on that route increases greatly.
--lose more
不理解~~~
5. B:When an airplane is taken out of service for maintenance, it is often repainted as well, and during the repainting no other maintenance work can be done on the plane.
P:airline officials are considering using a new nontoxic plastic film instead of paint.  The film takes just as long to apply as paint does, but many other maintenance tasks can be carried out at the same time.
C:reduce maintenance time
and the plastic can be used as long as the paint or longer
(A) Unlike paint, the film gives a milky tone to certain colors.
--irrelevant
(B) At the end of its useful life, the film can be removed much more quickly than paint can.
--right
(C) The film can be applied only by technicians who have received special training.
--weaken
(D) The metal exteriors of airplanes have to be protected from high temperatures and caustic chemicals such as exhaust gases.
--irrelevant
(E) Even at speeds considerably higher than the normal speed of a passenger jet, the film remains securely attached.
--irrelevant
作者: wangyiwen1005    时间: 2011-12-10 21:39
3. Modern navigation systems, which are found in most
of today’s commercial aircraft, are made with
low-power circuitry, which is more susceptible to
interference than the vacuum-tube circuitry found in
older planes. During landing, navigation systems
receive radio signals from the airport to guide the
plane to the runway. Recently, one plane with
low-power circuitry veered off course during landing,
its dials dimming, when a passenger turned on a
laptop computer. Clearly, modern aircraft navigation
systems are being put at risk by the electronic
devices that passengers carry on board, such as
cassette players and laptop computers.
Which one of the following, if true, LEAST
strengthens the argument above?

60s
BG: 新老定位系统对比
P: 新的省能源但会在降落时收到音波信号,有一次信号弱了因为飞机上有人打开了本本
C: 新定位系统因使用电子产品存在隐患

(A) After the laptop computer was turned off, the
plane regained course and its navigation
instruments and dials returned to normal.--直接support
(B) When in use all electronic devices emit
electromagnetic radiation, which is known to
interfere with circuitry.--support
(C) No problems with navigational equipment or
instrument dials have been reported on
flights with no passenger-owned electronic
devices on board.--弱support
(D) Significant electromagnetic radiation from
portable electronic devices can travel up to
eight meters, and some passenger seats on
modern aircraft are located within four meters
of the navigation systems.--support
(E) Planes were first equipped with low-power
circuitry at about the same time portable
electronic devices became popular.--基本无关,所以正确?猜的答案。。。
作者: fox0923    时间: 2011-12-10 23:28
3. Modern navigation systems, which are found in most
of today’s commercial aircraft, are made with
low-power circuitry, which is more susceptible to
interference than the vacuum-tube circuitry found in
older planes. During landing, navigation systems
receive radio signals from the airport to guide the
plane to the runway. Recently, one plane with
low-power circuitry veered off course during landing,
its dials dimming, when a passenger turned on a
laptop computer. Clearly, modern aircraft navigation
systems are being put at risk by the electronic
devices that passengers carry on board, such as
cassette players and laptop computers.
Which one of the following, if true, LEAST
strengthens the argument above?

60s
BG: 新老定位系统对比
P: 新的省能源但会在降落时收到音波信号,有一次信号弱了因为飞机上有人打开了本本
C: 新定位系统因使用电子产品存在隐患

(A) After the laptop computer was turned off, the
plane regained course and its navigation
instruments and dials returned to normal.--直接support
(B) When in use all electronic devices emit
electromagnetic radiation, which is known to
interfere with circuitry.--support
(C) No problems with navigational equipment or
instrument dials have been reported on
flights with no passenger-owned electronic
devices on board.--弱support
(D) Significant electromagnetic radiation from
portable electronic devices can travel up to
eight meters, and some passenger seats on
modern aircraft are located within four meters
of the navigation systems.--support
(E) Planes were first equipped with low-power
circuitry at about the same time portable
electronic devices became popular.--基本无关,所以正确?猜的答案。。。
-- by 会员 wangyiwen1005 (2011/12/10 21:39:44)



恩,我觉得是这样的,这个选项属于irrelevant,而其他四个或多或少都有support的成分。
作者: balapupu    时间: 2011-12-11 11:24
1.[40s]
P: modern system LP circuitry is more susceptible than the one in old system? recent days, one plan its signal is missing when one of its passenger use the laptop
C: the modern system has high risk than the old one because the use of the electronic devices.
Least support:
A:
(A) After the laptop computer was turned off, the
plane regained course and its navigation
instruments and dials returned to normal.-->support, no cause, no result.
(B) When in use all electronic devices emit
electromagnetic radiation, which is known to
interfere with circuitry.-->support, prove the evidence is reliable.
(C) No problems with navigational equipment or
instrument dials have been reported on
flights with no passenger-owned electronic
devices on board.-->support, no cause, no result
(D) Significant electromagnetic radiation from
portable electronic devices can travel up to
eight meters, and some passenger seats on
modern aircraft are located within four meters
of the navigation systems.-->support, prove the evidence is sufficient.
(E) Planes were first equipped with low-power
circuitry at about the same time portable
electronic devices became popular.-->weaken
作者: balapupu    时间: 2011-12-11 11:28
我觉得是不是E还会有点weaken的成分,因为electron device 是在modern 用的时候才开始广泛使用的,所以说old 用的时候,electron device 用的少,所以没有veer off, 如果用的话也会veer off 的~所以说新的不一定比旧的更容易造成危险。
作者: fox0923    时间: 2011-12-11 11:55
我觉得是不是E还会有点weaken的成分,因为electron device 是在modern 用的时候才开始广泛使用的,所以说old 用的时候,electron device 用的少,所以没有veer off, 如果用的话也会veer off 的~所以说新的不一定比旧的更容易造成危险。
-- by 会员 balapupu (2011/12/11 11:28:06)


没觉出来有weaken的成份阿,后面的bala说的意思我没明白,怎么还联系到old上面去了。
从选项来看, low-power circuitry at about the same time portable electronic devices became popular应该也就是说这两个发生的时间是一样的,是在electronic device get popular的时候。顶多就是表示一个时间的关系,没有什么weaken的含义阿~
木有明白bala的解释~

作者: mahaofei001    时间: 2011-12-11 17:49
无处不在的可爱糊糊~~~
作者: balapupu    时间: 2011-12-11 21:45
我觉得是不是E还会有点weaken的成分,因为electron device 是在modern 用的时候才开始广泛使用的,所以说old 用的时候,electron device 用的少,所以没有veer off, 如果用的话也会veer off 的~所以说新的不一定比旧的更容易造成危险。
-- by 会员 balapupu (2011/12/11 11:28:06)



没觉出来有weaken的成份阿,后面的bala说的意思我没明白,怎么还联系到old上面去了。
从选项来看, low-power circuitry at about the same time portable electronic devices became popular应该也就是说这两个发生的时间是一样的,是在electronic device get popular的时候。顶多就是表示一个时间的关系,没有什么weaken的含义阿~
木有明白bala的解释~
-- by 会员 fox0923 (2011/12/11 11:55:53)



我换个说法哈:看这个时间轴:vacuum-tube circuitry(old)------->low-power circuitry(new)+electronic
devices become popular.所以可以得出,以前vacuum-tube circuitry 使用的时候,electron devices 用的少,所以事故率低,如果用的和现在一样多的话,事故率也会增高的,所以我觉得会weaken。
作者: winghyy    时间: 2011-12-12 10:54
精炼题
57s-LEAST strengthen
One plane with low-power circuitry veered off course during landing, when a passenger turned on a
laptop computer. Clearly, modern aircraft navigation systems are being put at risk by the electronic
devices that passengers carry on board.
选E
(A) After the laptop computer was turned off, the plane regained course and its navigation
instruments and dials returned to normal.——strengthen (further explaination)
[when the cause is absent, the effect does not occur]
(B) When in use all electronic devices emit electromagnetic radiation, which is known to
interfere with circuitry.—— strengthen (further explaination)
[by showing that the data used to make the conclusion is accurate]
(C) No problems with navigational equipment or instrument dials have been reported on
flights with no passenger-owned electronic devices on board.—— strengthen
[when the cause is absent, the effect does not occur]
(D) Significant electromagnetic radiation from portable electronic devices can travel up to
eight meters, and some passenger seats on modern aircraft are located within four meters
of the navigation systems.——strengthen
[by showing that the data used to make the conclusion is accurate]
(E) Planes were first equipped with low-power circuitry at about the same time portable
electronic devices became popular. —— irrelevant

Modern navigation systems, which are found in most
of today’s commercial aircraft, are made with
low-power circuitry, which is more susceptible to
interference than the vacuum-tube circuitry found in
older planes. During landing, navigation systems
receive radio signals from the airport to guide the
plane to the runway. Recently, one plane with
low-power circuitry veered off course during landing,
its dials dimming, when a passenger turned on a
laptop computer. Clearly, modern aircraft navigation
systems are being put at risk by the electronic
devices that passengers carry on board, such as
cassette players and laptop computers.
Which one of the following, if true, LEAST
strengthens the argument above?
作者: 风无衣    时间: 2011-12-12 12:42
【精炼2-2】
least strengthen_58s
background information:modern navigation systems use low-power circuitry. old systems use vacuum-tube circuitry.navigation system could guide plane to runway when landing.
premise:recently a plane with low-power circuitry has a problem at landing when a passenger opened a laptop.
conclusion:modern navigation systems are at risk when passengers carry the electronic devices on board.

(A) After the laptop computer was turned off, the
plane regained course and its navigation
instruments and dials returned to normal.——support
(B) When in use all electronic devices emit
electromagnetic radiation, which is known to
interfere with circuitry.——support
(C) No problems with navigational equipment or
instrument dials have been reported on
flights with no passenger-owned electronic
devices on board.——support
(D) Significant electromagnetic radiation from
portable electronic devices can travel up to
eight meters, and some passenger seats on
modern aircraft are located within four meters
of the navigation systems.——support
(E) Planes were first equipped with low-power
circuitry at about the same time portable
electronic devices became popular.——irrelevant
作者: winghyy    时间: 2011-12-12 13:49
逻辑链
13. 50s-weaken
An unusually severe winter occured after an eruption.Therefore, major eruption leads to a cooler atomsphere.
Prephase: Another cause leaded to the severe winter.
选C [When A occurs, B does not occur]

14. 42s-assumption
Microwave ovens with built-in clock use more 45K electricity than those without the built-in clocks.
Prephase: In microwave ovens with no built-in clocks, there is no other devices that uses electricity.
选D

15.38s-weaken
Some airline companies attract consumers by offering very low fares on certain routes to drive competitors off those routes. However, this method is not profitable in the long run.Because after elimating the competitors, the airline company will charge high fares on the route to recoup the earlier losses, thus providing competitors a better opportunity to undercut the fares.
Prephase: The fares is not so low to make airlines loose money.
选E

16.47s-strengthen
Airplane maintenance always goes with repainting. The use of a new kind of nontoxic painting allow that the maintenace tasks can be carried out at the same time, in an attempt to reduce the maintenace time.
选B
作者: zz42050524    时间: 2011-12-12 16:59
2011/12/12 Least support
The airplane has a device with low signal to receive information from land. When passenger opens a laptop, the signal will diminish.
So the system will be at risk by the electricity devices carried by passenger.
Pre:
(A) After the laptop computer was turned off, the  support
plane regained course and its navigation
instruments and dials returned to normal.
(B) When in use all electronic devices emit     这个是答案:
electromagnetic radiation, which is known to
interfere with circuitry.
(C) No problems with navigational equipment or  support
instrument dials have been reported on
flights with no passenger-owned electronic
devices on board.
(D) Significant electromagnetic radiation from   support
portable electronic devices can travel up to
eight meters, and some passenger seats on
modern aircraft are located within four meters
of the navigation systems.
(E) Planes were first equipped with low-power support
circuitry at about the same time portable
electronic devices became popular.
这道题有个疑问,这个least是否定意思啊,这个就是找weaken或者无关。
作者: ugly5552000    时间: 2011-12-17 00:55
1/Background: the navigation systems which are installed on planes and play important role during the landing of planes are easily to be interfered.
Premise: recently, when a passage turned on a laptop computer during the landing of a plane, the navigation system happened to show some problem, so the electronic devices carried onto planes by passengers put the navigation at risk.
Prephrase: the problem during landing was by the storm weather condition.
E
A: it shows that the laptop computer has affects on navigation system
B: since all electronic devices emit radiation to interfere with navigation system, the argument is strengthened.
C: no electronic devices, no problems with the system, support
D: the system is in the radiation range of the electronic devices.
E: irrelevant, so least supports the argument
2/Background: once an severe winter occurred after the eruption of a volcano.
Premise: it is the eruption of volcano makes the atmosphere to become cooler.
Prephrase: after 1984, the volcano erupted several times, however, the atmosphere did not change after the eruption.
C
A: irrelevant
B: strong connection between eruption of volcano and rainy season supports the argument
C: after volcanic eruption, the temperature is still high, weakens the argument
D: irrelevant
E: supports
3/Background: when microwave ovens are not in use, the built-in clock still consume electricity. The clock each consume about 45 kilowatt-hours per year.
Premise: the microwave ovens without built-in clock consume 45 kilowatt-hours per year less than those with built-in clock.
Prephrase: the total hours per year the two kinds of microwave ovens are in use for are equal.
D
A: if it is true, the result of electrical consumption of built-in clock cannot be measured.
B: irrelevant
C: irrelevant
D: right, it rules out the factor which may trigger bias
E: irrelevant
4/Background: some airlines drive their competitors off the short routes by setting the fares of the routs lower the costs, but the strategy is not profitable in long routes.
Premise: once an airline implements the method, their attempts of recouping the earlier losses by charging high fares on that route for an extended period would provide competitors the chances to undercut the fares.
Prephrase: the high fares is accompanied with more service, which makes passengers feel worthwhile.
B
A: irrelevant
B: if there is competitor, the reduced price will maintain, then no opportunity is left to the competitors.
C: irrelevant, it merely depicts the promotion of tickets
D: irrelevant
E: irrelevant
5/Background: when an airplane is being repainted, no other maintenance work can be carried out on the same time.
Premise: the official plans to replace paint with a plastic film which not only sustain as long as the paint and allows other maintenance jobs to be carried out at the same time.
Prephrase: the new film is easy to acquire.
B
A: irrelevant
B: not only the film is equally easy-use as the paint, but also can it allow other maintenance tasks to be done at the same time
C: weaken
D: weaken
E: the point is “maintenance”, not security
作者: FB小贝    时间: 2012-4-28 16:45
【1】Background: Modern… to the runway.
Premise: Recently, one plane with
low-power circuitry veered off course during landing,
its dials dimming, when a passenger turned on a
laptop computer.
Conclusion:  Clearly, modern aircraft navigation
systems are being put at risk by the electronic
devices that passengers carry on board, such as
cassette players and laptop computers.
Least Strengthen(weaken): not the problem of electronic devices
A. Strengthen
B. Strengthen
C. Strengthen
D. Strengthen
E. Irrelevant ,correct
【2】Background: the continent was blanketed by a blue haze resulting from the eruption of the Laki Volcano in the European republic of Iceland in the summer of 1984.
Premise: An unusually severe winter occurred in Europe
Conclusion: major eruptions cause the atmosphere to become cooler than it would be otherwise.
Weaken: temperature rise after eruptions/severe winter is caused by other factors
A. Irrelevant
B. Irrelevant
C. correct
D. strengthen
E. Irrelevant—sea surface is not the same as air temperature
【3】Background: Most household appliances use electricity only when in use.  Many microwave ovens, however, have built-in clocks and so use some electricity even when they are not in use.  
Premise: The clocks each consume about 45 kilowatt-hours per year.
Conclusion: Therefore, households whose microwave oven has no built-in clock use 45 kilowatt-hours per year less, on average, than do comparable households whose microwave oven is otherwise similar but has a built-in clock.
Assumption: the two kinds of mo consume the same electricity except for the clock
A. Irrelevant
B. Irrelevant
C. Irrelevant
D. correct  no more plugged in clocks
E. Irrelevant
【4】Background: Some airlines allegedly reduce fares on certain routes to a level at which they lose money, in order to drive competitors off those routes.
Premise:  Once an airline successfully implements this method, any attempt to recoup the earlier losses by charging high fares on that route for an extended period would only provide competitors with a better opportunity to undercut the airline's fares.
Conclusion:  this method of eliminating competition cannot be profitable in the long run.  
Weaken: successfully implements will not cause losses
A. Irrelevant
B. Irrelevant
C. correct
D. Irrelevant
E. Strengthen
【5】Background: When an airplane is taken out of service for maintenance, it is often repainted as well, and during the repainting no other maintenance work can be done on the plane.  
Premise: The film takes just as long to apply as paint does, but many other maintenance tasks can be carried out at the same time.
Conclusion: reduce maintenance time
Strengthen: the film lasts at least as long as the paint
A. Irrelevant
B. Strengthen , correct
C. weaken
D. irrelevant
E. irrelevant
作者: baseboss    时间: 2012-6-4 21:05
1.Modern navigations will be interrupt by electronic device.
presume:
people like to open the device after landing.

(A) After the laptop computer was turned off, the
plane regained course and its navigation
instruments and dials returned to normal.
-->correct
(B) When in use all electronic devices emit
electromagnetic radiation, which is known to
interfere with circuitry.
-->irrelevant
(C) No problems with navigational equipment or
instrument dials have been reported on
flights with no passenger-owned electronic
devices on board.
-->irrelevant
(D) Significant electromagnetic radiation from
portable electronic devices can travel up to
eight meters, and some passenger seats on
modern aircraft are located within four meters
of the navigation systems.
-->irrelevant
(E) Planes were first equipped with low-power
circuitry at about the same time portable
electronic devices became popular.
-->irrelevant


2Become cooler is due to eruption
presume:
The evidence show the cooler is not due to eruption

(A) The cooling effect triggered by volcanic eruptions in 1985 was counteracted by an unusual warming of Pacific waters.
-->irrelevant
(B) There is a strong statistical link between volcanic eruptions and the severity of the rainy season in India.
-->irrelevant
(C) A few months after El Chichn's large eruption in April 1982, air temperatures throughout the region remained higher than expected, given the long-term weather trends.
-->irrelevant
(D) The climatic effects of major volcanic eruptions can temporarily mask the general warming trend resulting from an excess of carbon dioxide in the atmosphere.
-->weaken
(E) Three months after an early springtime eruption in South America during the late 19th century, sea surface temperatures near the coast began to fall.
-->support


3presume:
microwace ovens is always connect with the electricity.
(A) Households that do not have a microwave oven use less energy per year, on average, than do households that have a microwave oven.
-->irrelevant
(B) Microwave ovens with a built-in clock do not generally cost more to buy than microwave ovens without a built-in clock.
-->irrelevant
(C) All households that have a microwave oven also have either a gas oven or a conventional electric oven.
-->irrelevant
(D) Households whose microwave oven does not have a built-in clock are no more likely to have a separate electric clock plugged in than households whose microwave oven has one.
-->correct
(E) There are more households that have a microwave oven with a built-in clock than there are households that have a microwave oven without a built-in clock.
-->irrelevant

4compete by reduce fares is useless.
presuem:
reduce fares will let competitor leave the routes.

A) In some countries it is not illegal for a company to drive away competitors by selling a product below cost.
-->correct
(B) Airline executives generally believe that a company that once underpriced its fares to drive away competitors is very likely to do so again if new competitors emerge.
-->irrelevant
(C) As part of promotions designed to attract new customers, airlines sometimes reduce their ticket prices to below an economically sustainable level.
-->support
(D) On deciding to stop serving particular routes, most airlines shift resources to other routes rather than reduce the size of their operations.
-->irrelevant
(E) When airlines dramatically reduce their fares on a particular route, the total number of air passengers on that route increases greatly.
-->irrelevant

5 use films to instead of painting
presume:
good effort by using films

A) Unlike paint, the film gives a milky tone to certain colors.
-->weaken
(B) At the end of its useful life, the film can be removed much more quickly than paint can.
-->support correct
(C) The film can be applied only by technicians who have received special training.
-->weaken
(D) The metal exteriors of airplanes have to be protected from high temperatures and caustic chemicals such as exhaust gases.
-->irrelevant
(E) Even at speeds considerably higher than the normal speed of a passenger jet, the film remains securely attached.
-->support but no compare
作者: jetyxo    时间: 2012-6-26 16:50
1 33"  premise: the  data screen of airplane with new systems become dimming,when customers open electrical MP
          conclusion:new navigation system is put at risk when eletrical devices opened?
          LEAST strengthen?--->E
-----------------------------------------------------------
2 23" premise: In Europe, the winter became serious because Laki volcano broken
         conclusion: the volcano cause atmosphere become cooler and cooler
          weaken? there was other reason cause winter so serious
         A B D irrelevant ,E supports   answer: C
---------------------------------------------------------------------
3 35"  premise:  microwave oven with block-in colse MO1 use 45 khr more than that without block-in close MO2
          conclusion: households who use the later oven use 45 khr less  than household with block-in close ovens
          assumption:  household use MO2 does not need other close function as a block-in close
          answer
------------------------------------------------------------
4 33" background: an airline plan to reduce its price to drive other competitors off the route
        conclusion: this plan will make no profits in a long term,because once the airline increase its price, the competitors will come back
        weaken:B
--------------------------------------------------
5 28" background:when use  old method of painting , there are no other maintance work can be done,
         premise: in order to reduce the time need to maintain, the company plan to use new method of painting,which doesn't need to postpone other maintance when painting
       support? new methord won't waste time at other aspect
       answer:B
作者: meabh    时间: 2012-7-10 21:51
精炼(1) Time: 34s
(2) Category: Weaken
(3) Logical pattern:
Background: Modern NS are more susceptible to circuitry.
Premise: One recent plane was interfered with a laptop signals.
Conclusion: Modern NS put a risk at passengers’ safety by carry such devices
(4) Prephrase: Lack of representation
(5) Analysis:
(A) After the laptop computer was turned off, the plane regained course and its navigation instruments and dials returned to normal. -----strengthen
(B) When in use all electronic devices emit electromagnetic radiation, which is known to interfere with circuitry.-----strengthen
(C) No problems with navigational equipment or instrument dials have been reported on flights with no passenger-owned electronic devices on board.------strengthen
(D) Significant electromagnetic radiation from portable electronic devices can travel up to eight meters, and some passenger seats on modern aircraft are located within four meters of the navigation systems.----strengthen
(E) Planes were first equipped with low-power circuitry at about the same time portable electronic devices became popular.-------irrelevant

13.(1) Time: 29s
(2) Category: Weaken
(3) Logical pattern:
Premise: Cold winter occurred after eruptions
Conclusion: Eruptions caused cold weather
(4) Prephrase: Cold weather caused eruptions.
(5) Analysis: C

14(1) Time: 30s
(2) Category: Assumption
(3) Logical pattern:
Background: Electronic appliances use electricity when they are in use.
Premise: Many ovens have built-in clocks which worked even ovens are not in use.
Conclusion: Ovens with built-in clocks consumed more electricity.
(4) Prephrase: Other ovens don’t have other clocks which may consume electricity as well.
(5) Analysis: D

15(1) Time: 23s
(2) Category: Weaken
(3) Logical pattern:
Background:
Premise: Some airlines charged less to rule out competitors.
Conclusion: When they raised price new competitors will emerge.
(4) Prephrase: Airlines have something else different to avoid such situation.
(5) Analysis: B

16(1) Time: 36s
(2) Category: Strengthen
(3) Logical pattern:
Background: Paint waste time to do maintenance work.
Premise: A new film takes same time to apply but can do other work at the same time.
Conclusion: New film can replace the older paint.
(4) Prephrase: Film can last as long as paint.-----------other merits
(5) Analysis: B
作者: Donts    时间: 2012-7-21 10:26
1.计时48''premisene plane with L veered off course during landing because a passenger turn on a laptop.
conclusion:modern aircraft navigation systems are being at risk by electronic devices that passengers carry on board.
问题有点奇异
选择A
2.计时29''
premise:a unusual severe winter occurred after E was covered by B resulting from eruption of V in the summer.
conclusion:the eruption causes the atmosphere to become cooler.
推测:别的原因导致,切断eruption和unusual temprature关系
选择C。D有点纠结
3.计时43''
premise:built in clock use 45 k/h per year
conclusion:household with built-in clock use 45k/h more per year than those without built-in clock
选择D
4.计时42''
premise:some airline reduce fares on a certain route to a level at which they lose money to eliminate competition. once they succeed, they will charge more fares, which give their competitor opportunities to undercut the airlines' fares.
conclusion:the method of eliminating competition cannot be profitable in the long run.
选择E
5.计时43''
plan:use a new thing instead of paint so that maintenance and the thing can be done at the same time.
goal:to reduce the maintenance time.
选择B
订正1.E  4.B
问题:4为什么选B,2的D?
补充: Least works like Except in question stems
作者: wanggang0411    时间: 2012-8-30 15:46
47
Premise: low-power circuity are more susceptible to interference than the old one.
Conclusion: morden aircraft navigation system are in the risk.
E
E

25
Premise: A severe winter occrured in winter.
Premise: the continent is blanketed by haze.
Premise: the haze is resulted from the eruption of a volcano.
Conclusion: the eruption of a volcano caused the atmosphere to become cooler.

Fallacious causality
The author commited a fallacy of causal oversimplication.
The line of the reason is that because the haze appeared before the severe winter occured,
the former event is responsible for the later one.
However, the fact that haze appeared coincides with the severe winter occured
does not necessarily prove that the former event caused the later one.
In fact, it is possible that other climate factors caused the occurrance of severe winter.
Therefore, this is a fallacious reasoning unless other possible explainations have been
concerned and ruled out.

A: this answer eliminate the climatic effects of the eraption of volcano.
Answer A is a supporter, since it confirmed that the eraption really caused the atmosphere cooler.
Answer C is a weakener, since it indicates that eraption caused a effect opposite to the effect supposed in the argument.
Answer D is irrelevant, since the eraption can mask the general warming trend resulting from another reason.



Premise: Microwave with built-inclock consume 45 kh per years.
Conclusion: Households whose oven doesn't have built-in clock use 45 kh per year less
than do comparable houseshoulds whose oven has built-in clock.
??
D: no seperated electric clock.

35
Premise: Some airlines reduce the fare to drive competitors off those routes.
Premise: This method can not be profitable in the long run.
Conclusion: Any attempt to recoup the earlier losses will provide competitors with
a opportunity to undercut the airline's fares.
Weaken: It's quite difficalut for competitors to rebuild a routes.
B: maybe right
B

36
Premise: To reduce the maintenance time, airline officials are considering using
a film to do the paint.
Conclusion: Durning the paint, other maintenance task can be done.
Support:
B: this is another advantage of film, which can reduce the time cost.
B
作者: 我心匪席    时间: 2012-9-13 12:53
Time: 42”
Background Information:  Modern NS, made with low-power circuitry, is more susceptible to interference than VT circuitry in older planes. During landing, NS receive radio signals from airport to guide the plan to runway.
Premise: One modern plan veered off during landing, when a passenger turned on a laptop computer.
Conclusion: Modern aircraft NS are being put risk by the electronic devices that passengers carry on board.
A-----support
B-----support
C-----support
D-----support
E-----right irrelevant
===============================================
Time: 22”
Premise: Severe winter occurred in Europe after continent was blanketed by a blue haze resulting from the eruption of the Volcano in 1984.
Conclusion: Major eruptions cause the atmosphere to become cooler than it would be otherwise.
Prephase: Other factor causes severe winter.
A-----support
B-----support
C-----support(right weaken)
D-----right weaken
E-----irrelevant
===============================================
Time: 30”
Background Information: Most household appliances use electricity only when in use.
Premise: Microwave has built-in clocks use some electricity.
Conclusion: Microwave built-in clocks use 45 kilowatt-hours per year than similar microwave without built-in clocks.
Prephase: People use microwave ovens built-in clocks as much as people use similar microwave ovens without built-in clocks.
A-----irrelevant
B-----irrelevant
C-----irrelevant
D-----right assumption
E-----irrelevant
================================================
Time: 32”
Background Information: Some airlines reduce fares on certain routes to drive competitors off these routes.
Premise: To recoup the earlier losses by charging high fares on that route for an extended period would only provide competitors with a better opportunity to undercut the airline’s fares.
Conclusion: This method of eliminating competition cannot be profitable in the long run.
Prephase: People use microwave ovens built-in clocks as much as people use similar microwave ovens without built-in clocks.
A-----irrelevant
B-----right weaken
C-----irrelevant
D-----irrelevant
E-----support

感谢楼主整理~
作者: srafcatt    时间: 2012-10-23 01:47
精炼:weaken 26s
navigation system 导航系统
be made with 由...做成的
be susceptible to 对...敏感的
vacuum-tube circuitry 真空管电路
veer off course 偏离跑道
allow...to be put at risk使...处于危险当中
put...at higher/greater risk
electronic devices电子设备
on board 在火车,飞机,船,汽车上
cassette players 卡式录音机
laptop computers便携式计算机
electromagnetic radiation 电磁辐射
fact:aircraft's navi sys are more susceptible to inference than old ones
     when landing, sys recieve signal to guide plane to runway
premise:a laptop computer caused dials dimming & veer off courses
conclusion:elec device interfere with navi sys
If you want to strenghten,you must say the elec device can interfere dials just like computers
answer:E
A this answer strenghten the argument by confirming that everything is normal without the interference.
B this answer strenghten the argument
C this answer also strenghtens the argument by eliminating other possibilities that might cause the same consequence.
D this one makes sure that the radiation from laptop surely can infect the system
E correct answer.maybe it has some relationship between these two phenomenon,but it didn't mention.
-------------------------------------------------------------------------------------逻辑链
1.weaken 20s
premise:after haze in summer,winter came
conclusion:eruptions cause cooler atmosphere.
if winter is cold just because of other reason,then the argument is weaken successfully.
answer:A
A this answer weakens the argument as it shows eruptions are ineffective.
B so what?India has nothing to do with European
C this answer offers irrelevant information to us
D this question stem is talking about severe winter,not cooler summer.
E the answer didn't mention the relationship between the seawater temperature and the severe winter.
2.assumption 26s
premise: microwave oven's build-in clock consume electricity
conclusion: microwave oven with a clock consume electricity just the same amount
my assumptionnly the clock in the oven keeps consuming electricity even the oven was not being used.
answer
A irrelevant information
B a shell game. the question stem is talking about the electricity not money
C irrelevant information
D correct
E it has nothing to do with the households who don't have microwave oven because those people are not concluded
3.weaken 23s
reduce fare---drive competitors off
while not in the long run---the competitor has another opportunity
what if the competitor has broken out?
answer:B
A irrelevant information
B correct
Cirrelevant information
D irrelevant information
E this is not related to the topic,in fact the more the passengers are the more money they lose
4.strenghten 27s
airplane maintenance--repainted--no other work can be done
premise:plastic film  as long as the paint to apply and makes work done at the same time
conclusion:time saved
the plastic can prevent the dirty things from splitting on the plane
answer:B
A not convincing
B correct answer,save time more effectively
C weaken the argument,because it might need time to train them
D irrelevant information
E irrelevant information
作者: srafcatt    时间: 2012-10-23 02:13
纠错:逻辑链第一题:
注意:此题的结论是:Thus, it is evident that major eruptions cause the atmosphere to become cooler than it would be otherwise.
选A错,因为要反驳的逻辑是eruption cause cooler atmosphere,而这个选项中的unusual暗示我们这是个特例。特例是不足为信的。
(C) A few months after El Chichn's large eruption in April 1982, air temperatures throughout the region remained higher than expected, given the long-term weather trends.
这个是提供了结论中的条件,但是得到了与结论相反的结果,而且并非特例,因此是正确选项。

本题启示:要仔细读题。这道题其实weaken 中的举反例反驳题。
weaken一个逻辑,可以从前提下手,说前提是不存在的
                             也可以从实施的过程中下手,说这是不可行的
                             还有一种就是从结论下手,即举反例。举反例要注意的是不能举非常特殊的很少出现的例子。因为极端情况是不具有可信度的。
作者: srafcatt    时间: 2012-10-23 02:19
学学人家我心匪席,我也说一句:   感谢楼主整理~辛苦了。
作者: oytt1111    时间: 2012-10-23 19:48
3 Time: 60s
Premise: Modern navigation system are low-power circuitry.
Premise: During landing, NS receive signals from airport.
Premise: one plane with low-pc veered off because of passenger’s computer.
Conclusion: modern NS are at risk because of passengers’ e device.
Type: strengthen X
Answer: E. It’s irrelevant while others are strengthen.

13 Time: 28s
Premise: severe winter occurred in E because of eruption of L V in 1984.
Conclusion: it’s evident that major eruptions cause cooler atmosphere.
Type: weaken
Prephrase: some other reasons occurred in 1984
        Facts with same reason but different result  
Answer: C

14 Time: 47s
Premise: microwave ovens with clock consume 45 kh per year.
Conclusion: microwave without clock will save 45 kh per year than those with clocks.
Type: assumption
Prephrase: other conditions are equally same
Answer: B

15 Time: 65s
Premise: Some airlines reduce fares to a level losing money to drive competitors off those routes.
Premise: this method cannot be profitable in the long run
Conclusion: once they succeed, their attempt to recoup losses will cause competitor’s opportunity.
Type: weaken
Answer: D

16 Time: 44s
Premise: other works can not be done during repainting.
Premise: nontoxic film can replace of painting while other works can be done the same time
Answer: E
作者: penguinsdz    时间: 2012-10-24 02:30
标题: 10.23+【2-2】
10.23+2-2

1. Modern navigation systems, which are found in most of today’s commercial aircraft, are made with low-power circuitry, which is more susceptible to interference than the vacuum-tube circuitry found in older planes. During landing, navigation systems receive radio signals from the airport to guide the plane to the runway. Recently, one plane with low-power circuitry veered off course during landing, its dials dimming, when a passenger turned on a laptop computer. Clearly, modern aircraft navigation systems are being put at risk by the electronic devices that passengers carry on board, such as cassette players and laptop computers.
Which one of the following, if true, LEASTstrengthens the argument above?
(A) After the laptop computer was turned off, the plane regained course and its navigation instruments and dials returned to normal.
(B) When in use all electronic devices emit electromagnetic radiation, which is known to interfere with circuitry.
(C) No problems with navigational equipment or instrument dials have been reported on flights with no passenger-owned electronic devices on board.
(D) Significant electromagnetic radiation from portable electronic devices can travel up to eight meters, and some passenger seats on modern aircraft are located within four meters of the navigation systems.
(E) Planes were first equipped with low-power circuitry at about the same time portable electronic devices became popular.

Background Information:
Modern navigation systems, which are found in most of today’s commercial aircraft, are made with low-power circuitry, which is more susceptible to interference than the vacuum-tube circuitry found in older planes.
Premise:
Recently, one plane with low-power circuitry veered off course during landing, its dials dimming, when a passenger turned on a laptop computer.

Conclusion:
Clearly, modern aircraft navigation systems are being put at risk by the electronic devices that passengers carry on board, such as cassette players and laptop computers.

选项分析:E



2. An unusually severe winter occurred in Europe after the continent was blanketed by a blue haze resulting from the eruption of the Laki Volcano in the European republic of Iceland in the summer of 1984. Thus, it is evident that major eruptions cause the atmosphere to become cooler than it would be otherwise.
Which of the following statements, if true, most seriously weakens the argument above?
(A) The cooling effect triggered by volcanic eruptions in 1985 was counteracted by an unusual warming of Pacific waters.
(B) There is a strong statistical link between volcanic eruptions and the severity of the rainy season in India.
(C) A few months after El Chichn's large eruption in April 1982, air temperatures throughout the region remained higher than expected, given the long-term weather trends.
(D) The climatic effects of major volcanic eruptions can temporarily mask the general warming trend resulting from an excess of carbon dioxide in the atmosphere.
(E) Three months after an early springtime eruption in South America during the late 19th century, sea surface temperatures near the coast began to fall.

Premise:
An unusually severe winter occurred in Europe after the continent was blanketed by a blue haze resulting from the eruption of the Laki Volcano in the European republic of Iceland in the summer of 1984. Conclusion:
Thus, it is evident that major eruptions cause the atmosphere to become cooler than it would be otherwise.

选项分析:C



3. Most household appliances use electricity only when in use. Many microwave ovens, however, have built-in clocks and so use some electricity even when they are not in use. The clocks each consume about 45 kilowatt-hours per year. Therefore, households whose microwave oven has no built-in clock use 45 kilowatt-hours per year less, on average, than do comparable households whose microwave oven is otherwise similar but has a built-in clock.
Which of the following is an assumption on which the argument depends?
(A) Households that do not have a microwave oven use less energy per year, on average, than do households that have a microwave oven.
(B) Microwave ovens with a built-in clock do not generally cost more to buy than microwave ovens without a built-in clock.
(C) All households that have a microwave oven also have either a gas oven or a conventional electric oven.
(D) Households whose microwave oven does not have a built-in clock are no more likely to have a separate electric clock plugged in than households whose microwave oven has one.
(E) There are more households that have a microwave oven with a built-in clock than there are households that have a micr
owave oven without a built-in clock.

Background Information:
Most household appliances use electricity only when in use.

Premise:
Many microwave ovens, however, have built-in clocks and so use some electricity even when they are not in use.

The clocks each consume about 45 kilowatt-hours per year.
Conclusion:
Therefore, households whose microwave oven has no built-in clock use 45 kilowatt-hours per year less, on average, than do comparable households whose microwave oven is otherwise similar but has a built-in clock.

推测(Prephrase: mentally formulate your answer to the question stem)

Microwave oven that does not have a built-in clock usually have no other separate electric clock devises.
选项分析:D



4. Some airlines allegedly reduce fares on certain routes to a level at which they lose money, in order to drive competitors off those routes. However, this method of eliminating competition cannot be profitable in the long run. Once an airline successfully implements this method, any attempt to recoup the earlier losses by charging high fares on that route for an extended period would only provide competitors with a better opportunity to undercut the airline's fares.
Which of the following, if true, most seriously weakens the argument?
(A) In some countries it is not illegal for a company to drive away competitors by selling a product below cost.
(B) Airline executives generally believe that a company that once underpriced its fares to drive away competitors is very likely to do so again if new competitors emerge.
(C) As part of promotions designed to attract new customers, airlines sometimes reduce their ticket prices to below an economically sustainable level.
(D) On deciding to stop serving particular routes, most airlines shift resources to other routes rather than reduce the size of their operations.
(E) When airlines dramatically reduce their fares on a particular route, the total number of air passengers on that route increase greatly.

Background Information:
Some airlines allegedly reduce fares on certain routes to a level at which they lose money, in order to drive competitors off those routes.
Premise:
Once an airline successfully implements this method, any attempt to recoup the earlier losses by charging high fares on that route for an extended period would only provide competitors with a better opportunity to undercut the airline's fares.
Conclusion:

However, this method of eliminating competition cannot be profitable in the long run.

推测(Prephrase: mentally formulate your answer to the question stem)

If there is competitor, the reduced price will maintain, so no opportunity is left to the competitors.

选项分析:B



5. When an airplane is taken out of service for maintenance, it is often repainted as well, and during the repainting no other maintenance work can be done on the plane. In order to reduce maintenance time, airline officials are considering using a new nontoxic plastic film instead of paint. The film takes just as long to apply as paint does, but many other maintenance tasks can be carried out at the same time.
Which of the following, if true, is further evidence that using the film will help the airline officials achieve their goal?
(A) Unlike paint, the film gives a milky tone to certain colors.
(B) At the end of its useful life, the film can be removed much more quickly than paint can.
(C) The film can be applied only by technicians who have received special training.
(D) The metal exteriors of airplanes have to be protected from high temperatures and caustic chemicals such as exhaust gases.
(E) Even at speeds considerably higher than the normal speed of a passenger jet, the film remains securely attached.

Background Information:
When an airplane is taken out of service for maintenance, it is often repainted as well, and during the repainting no other maintenance work can be done on the plane.

Premise:
The film takes just as long to apply as paint does, but many other maintenance tasks can be carried out at the same time.

Conclusion:
The airline officials can achieve their goal.
选项分析:B




作者: zxppx    时间: 2013-1-17 16:39
3. Modern navigation systems, which are found in most
of today’s commercial aircraft, are made with
low-power circuitry, which is more susceptible to
interference than the vacuum-tube circuitry found in
older planes. During landing, navigation systems
receive radio signals from the airport to guide the
plane to the runway. Recently, one plane with
low-power circuitry veered off course during landing,
its dials dimming, when a passenger turned on a
laptop computer. Clearly, modern aircraft navigation
systems are being put at risk by the electronic
devices that passengers carry on board, such as
cassette players and laptop computers.

The logic chain of the argument is that because when a passenger turned on a laptop computer, on plane with low-power circuitry veered off course during landing its dials dimming, we can conclude that modern aircraft navigation systems are being put at risk by the electronic devices that passengers carry on board, such as cassette players and laptop computers. In short, two things happened simultaneously, one caused the other. In order to weaken the argument, we need to break the causality. Perhaps, the relationship is reversed, or even no related to each other.
Which one of the following, if true, LEAST
strengthens the argument above?

(A) After the laptop computer was turned off, the
plane regained course and its navigation
instruments and dials returned to normal.

This answer choice actually strengthens the causality.

When the cause is absent, the effect does not occur.
(B) When in use all electronic devices emit
electromagnetic radiation, which is known to
interfere with circuitry.

This answer choice reveals that using electronic devices can interfere with circuitry.

This answer strengthens the argument by showing that the data used to demonstrate the causality is accurate.
(C) No problems with navigational equipment or
instrument dials have been reported on
flights with no passenger-owned electronic
devices on board.

From the opposite, we can see that this answer choice strengthens the argument in some degree.

This answer strengthens the argument by showing that when the cause is absent, the effect does no occur.
(D) Significant electromagnetic radiation from
portable electronic devices can travel up to
eight meters, and some passenger seats on
modern aircraft are located within four meters
of the navigation systems.

It still points out the radiation can inference with the navigation systems.

This answer strengthens the argument by showing that the data used to make the conclusion is accurate.
(E) Planes were first equipped with low-power
circuitry at about the same time portable
electronic devices became popular.

In this case, it does not have effect on the causality, since the time cannot prove the causal relationship between them. BA



13.(25752-!-item-!-188;#058&002668)
An unusually severe winter occurred in Europe after the continent was blanketed by a blue haze resulting from the eruption of the Laki Volcano in the European republic of Iceland in the summer of 1984. Thus, it is evident that major eruptions cause the atmosphere to become cooler than it would be otherwise.
Which of the following statements, if true, most seriously weakens the argument above?

A) The cooling effect triggered by volcanic eruptions in 1985 was counteracted by an unusual warming of Pacific waters.

Since the Pacific waters were unusual warming, the example is unrepresentative.

It still reveals that eruptions can cause the atmosphere to become cooler than it would be.
(B) There is a strong statistical link between volcanic eruptions and the severity of the rainy season in India.

Actually, this answer choice strengthens the argument.
(C) A few months after El Chichn's large eruption in April 1982, air temperatures throughout the region remained higher than expected, given the long-term weather trends.

The same to A.

In this case, it points out that the large eruption can cause the air temperatures throughout the region higher than expected. Please read carefully.
(D) The climatic effects of major volcanic eruptions can temporarily mask the general warming trend resulting from an excess of carbon dioxide in the atmosphere.

In this case, it cast doubt on the argument, since the general warming trend resulting from an excess o f carbon dioxide in the atmosphere can be caused by major volcanic eruptions. BA

I missed the meaning of “mask” and did not read carefully about what “resulting from”.

In fact, this answer choice strengthens the argument, rather than weakens.
(E) Three months after an early springtime eruption in South America during the late 19th century, sea surface temperatures near the coast began to fall.

Actually, this answer choice strengthens the argument.



14.(25938-!-item-!-188;#058&002908)
Most household appliances use electricity only when in use. Many microwave ovens, however, have built-in clocks and so use some electricity even when they are not in use. The clocks each consume about 45 kilowatt-hours per year. Therefore, households whose microwave oven has no built-in clock use 45 kilowatt-hours per year less, on average, than do comparable households whose microwave oven is otherwise similar but has a built-in clock.
Which of the following is an assumption on which the argument depends?

(A) Households that do not have a microwave oven use less energy per year, on average, than do households that have a microwave oven.

Shell Game: The comparison between households that do not have a microwave oven and households that have a microwave oven is irrelevant.
(B) Microwave ovens with a built-in clock do not generally cost more to buy than microwave ovens without a built-in clock.

The prices of them are outside the scope of the argument.
(C) All households that have a microwave oven also have either a gas oven or a conventional electric oven.

Who knows? It is irrelevant.
(D) Households whose microwave oven does not have a built-in clock are no more likely to have a separate electric clock plugged in than households whose microwave oven has one.

Yes, in this case, this answer choice rules out the hole in the argument, thus strengthening the logic chain. BA
(E) There are more households that have a microwave oven with a built-in clock than there are households that have a microwave oven without a built-in clock.

This is outside the scope of the argument.



15.(25986-!-item-!-188;#058&002914)
Some airlines allegedly reduce fares on certain routes to a level at which they lose money, in order to drive competitors off those routes. However, this method of eliminating competition cannot be profitable in the long run. Once an airline successfully implements this method, any attempt to recoup the earlier losses by charging high fares on that route for an extended period would only provide competitors with a better opportunity to undercut the airline's fares.
Which of the following, if true, most seriously weakens the argument?

(A) In some countries it is not illegal for a company to drive away competitors by selling a product below cost.

How about the other countries?
(B) Airline executives generally believe that a company that once underpriced its fares to drive away competitors is very likely to do so again if new competitors emerge.

It seems to be a contender. I choose it! BA

it states that other airlines are likely to continue to stay away, even after the big mean price-cutting airline raises its prices back up. (if their executives believe that 'big air' will simply lower its prices again if they try to wedge back into the market, then they'll stay out.) from Manhattan
(C) As part of promotions designed to attract new customers, airlines sometimes reduce their ticket prices to below an economically sustainable level.


The promotions designed to attract new customers are outside the scope of the argument, which talks about the aim to drive competitors off.
(D) On deciding to stop serving particular routes, most airlines shift resources to other routes rather than reduce the size of their operations.

Deciding to stop serving particular routes is outside the scope of the argument.
(E) When airlines dramatically reduce their fares on a particular route, the total number of air passengers on that route increases greatly.

If competitors use this strategy, they can benefit from it, thus making the argument much less valid.



16.(26678-!-item-!-188;#058&003277)
When an airplane is taken out of service for maintenance, it is often repainted as well, and during the repainting no other maintenance work can be done on the plane. In order to reduce maintenance time, airline officials are considering using a new nontoxic plastic film instead of paint. The film takes just as long to apply as paint does, but many other maintenance tasks can be carried out at the same time.
Which of the following, if true, is further evidence that using the film will help the airline officials achieve their goal?

(A) Unlike paint, the film gives a milky tone to certain colors.

The colors are irrelevant.
(B) At the end of its useful life, the film can be removed much more quickly than paint can.

At first glance, the answer choice seems to be irrelevant, but if the film can be removed much more quickly than paint can, the time to maintenance can be reduced. BA
(C) The film can be applied only by technicians who have received special training.

In some degree, this answer choice weakens the argument, since the workers needs some special training, which may increase the whole time for maintenance.
(D) The metal exteriors of airplanes have to be protected from high temperatures and caustic chemicals such as exhaust gases.

In this case, other work may not be conducted, considering the influence to the metal exteriors of airplanes.
(E) Even at speeds considerably higher than the normal speed of a passenger jet, the film remains securely attached.

This is out of the scope of the argument.

作者: pennyz    时间: 2013-3-4 18:24
3:00s
least stregthen
conclusion: the low - navagation is affected by electronic devices
stregthen:a,b,c
c is least
with no electronic devices ,no accidents
can not really approve
??
作者: pennyz    时间: 2013-3-4 18:41
2:36s
c
weaken
phenomenon:severe weather come after the eruption
conclusion: eroption can cause severe weather
rephrase: the t may not decrease after the eruption
s
1:40s
b
assumption
backgroung:2 type with clock and without a clock
premise:clock cost ...electricity
conclusion:clock cost more ...than no clock
rephrase: no other differences between no clock and clock

1:38s
weaken
conclusion: may not be profitable in the long run
          since once they raise the price ,new comer will get this airline with low price
rephrase: may not be any new comers
d
1:52s
support
premise: film use same time ,but can allow different maintain works
conclusion: can reduce the maintain time
rephrase: other information about film time saving activity
b
作者: Feelalive    时间: 2013-7-15 14:45
Feelalive到此一游
作者: meckyona    时间: 2013-9-4 22:35
B: modern navigation systems are made of low-power circuitry,which is more susceptible to
interference than the vacuum-tube circuitry found in older planes.  During landing, navigation systems
receive radio signals from the airport to guide the plane to the runway.

Pne plane with low-power circuitry had problems when landing when a passenger turned on a laptop.

C: modern aircraft navigation system is harmed by electronic devices psaaengers carried on board

least strenghthe----削弱

可能是当电子设备开启才会影响

A 提供证据

B 加强

C 削弱..right one 加强
D 支持原文观点,

E irrelevent right one

B: unusual severe winter in Europe
P:blue haze from eruption of a volcano in Europe
C:eruptions cause air to become cooler

问weaken
可能有其他原因导致冷

A:。。。
B:inrevelent
C:对 (不确定QAQ
D:有削弱,但是暂时的
E 加强了论点

B: most microwave have clocks and they consume 45 per year
C: microwave without clocks consume less than 45 a year

问哪个假设可以得出此结论(重点在有无时钟花费的能量

A:是microwave有无时钟的比较,而不是有没有microwave的比较
B:与microwave的价格无关
C:与其他的工具无关
D:right
E不是比种类的数量


B:some airlines reduce fares on routes in order to compete with competitors
Pnce an airline implements this method,  any attempt to recoup the earlier losses by charging high fares on that route for an extended period would only provide competitors with a better opportunity to undercut the airline's fares.
C:this method can't be profit in long run

问削弱,如果公司不需要再提高价格就能盈利

A:这是加强
B:与问题无关
C:这也是加强。。而且题中已经提到
D:无关
E:对
正确答案是B,做这个题我重点关注在降价的公司上而忽略了题目的重点还有怎么驱逐竞争者上......但这个也太绕了
B:when repaining of a plane, it can;t be mantaince as well,thus cost lots of time
P:a new film can be used and take the same time while lots of mantainance works can be carried out
问支持的证据 goal是缩短维护时间

A:无关
B:对
C:无关
D:无关
E:无关。。。
作者: Elisha728    时间: 2013-9-5 07:27
6'19''
ECDEE
作者: Elisha728    时间: 2013-9-7 11:24
15, 25986-!-item-!-188;#058&002914)
Some airlines allegedly reduce fares on certain routes to a level at which they lose money, in order to drive competitors off those routes.  However, this method of eliminating competition cannot be profitable in the long run.  Once an airline successfully implements this method, any attempt to recoup the earlier losses by charging high fares on that route for an extended period would only provide competitors with a better opportunity to undercut the airline's fares.
Which of the following, if true, most seriously weakens the argument?
(A) In some countries it is not illegal for a company to drive away competitors by selling a product below cost. Irrelevant
(B) Airline executives generally believe that a company that once underpriced its fares to drive away competitors is very likely to do so again if new competitors emerge-correct answer
If there is competitor, the reduced price will maintain, then no opportunity is left to the competitors-from a DN.
(C) As part of promotions designed to attract new customers, airlines sometimes reduce their ticket prices to below an economically sustainable level. Already stated in the passage
(D) On deciding to stop serving particular routes, most airlines shift resources to other routes rather than reduce the size of their operations. Irrelevant
(E) When airlines dramatically reduce their fares on a particular route, the total number of air passengers on that route increases greatly.
This is the most attractive wrong answer. The conclusion is about the consequences of airlines’ attempt to recoup the earlier losses by charging high fares. So what happens during the promotions period is irrelevant.
作者: Elisha728    时间: 2013-9-7 11:32
19.(26678-!-item-!-188;#058&003277)
When an airplane is taken out of service for maintenance, it is often repainted as well, and during the repainting no other maintenance work can be done on the plane.  In order to reduce maintenance time, airline officials are considering using a new nontoxic plastic film instead of paint.  The film takes just as long to apply as paint does, but many other maintenance tasks can be carried out at the same time.

Which of the following, if true, is further evidence that using the film will help the airline officials achieve their goal?

(A) Unlike paint, the film gives a milky tone to certain colors. irrelevant
(B) At the end of its useful life, the film can be removed much more quickly than paint can.-thus save time, correct answer.
(C) The film can be applied only by technicians who have received special training. irrelevant
(D) The metal exteriors of airplanes have to be protected from high temperatures and caustic chemicals such as exhaust gases. irrelevant
(E) Even at speeds considerably higher than the normal speed of a passenger jet, the film remains securely attached. Irrelevant, the goal is about time-saving, security is out of scope.
Analysis:
The goal is to reduce maintenance time. Find out an option that prove that this film can really save time.

作者: jessie92730    时间: 2013-9-22 11:30
【offer大冒险】2-2
精练
BG: modern one is more susceptible to interference than the older one
P: passenger – laptop- a plane with modern one veered of course
C: modern one more risky
推测答案:与前提结论都无关的选项
答案:E (与前提结论无关)

逻辑链:CDBB

作者: stellagee    时间: 2013-9-22 12:07
offer大冒险
BG:modern navigation system with low-power circuitry>older
During landing, navigation system receive signal from plane.
P: one plane with modern navigation system had an accident because of a laptop computer
C:MNS are being put at risk by the electronic devices.

A:least strengthen

猜测Strengthen: 得知laptop和许多electronic devices的影响相同

(A) After the laptop computer was turned off, the
plane regained course and its navigation
instruments and dials returned to normal.
证明是laptop的影响
(B) When in use all electronic devices emit
electromagnetic radiation, which is known to
interfere with circuitry.
在使用时 所有electronic devices都发出公认影响circuitry的ER
(C) No problems with navigational equipment or
instrument dials have been reported on
flights with no passenger-owned electronic
devices on board.
没electronic devices的没问题
(D) Significant electromagnetic radiation from
portable electronic devices can travel up to
eight meters, and some passenger seats on
modern aircraft are located within four meters
of the navigation systems.
ER可达到8米,而electronic devices离4米
(E) Planes were first equipped with low-power
circuitry at about the same time portable
electronic devices became popular.

作者: sakurasong    时间: 2013-9-22 20:41
CCDEB-ECDDB
B:Modern navigation system is more susceptible to interference than CT.
P1: during landing, NS receive radio signals
P2: One plane was influenced when a passenger turned on a computer
C:NS are being put at risk

E,irrelevant


B:Most household appliances use E only when in use
P1: Many MOs have build-in clocks and so use some E when they r not in use
P2: The clocks comsume about 45 KH per year
C: MO that has no built-in close use 45 KH less

作者: lyrsilvia    时间: 2013-9-22 20:50
E CDBB
1. 48'
P: one plane with low-power circuitry veered off course during landing, its dials dimming, when a passenger turned on a laptop computer.
C:  modern aircraft navigation systems are being put at risk by the electronic devices that passengers carry on board, such as cassette players and laptop computers.
LEAST STRENGTHEN:
答案:E
(A) After the laptop computer was turned off, the plane regained course and its navigation instruments and dials returned to normal.-----------support. means the laptop is the reason
(B) When in use all electronic devices emit electromagnetic radiation, which is known to interfere with circuitry.-----------support, means the electronic devices are to be blamed
(C) No problems with navigational equipment or instrument dials have been reported on flights with no passenger-owned electronic devices on board.-----------support the electronic devices is the cause
(D) Significant electromagnetic radiation from portable electronic devices can travel up to eight meters, and some passenger seats on modern aircraft are located within four meters of the navigation systems.----------still means the electronic devices are the causes
(E) Planes were first equipped with low-power circuitry at about the same time portable electronic devices became popular.----------CORRECT. when the planes were equipped does not explain the electronic devices affect.

2. 17'
P: An unusually severe winter occurred in Europe after the continent was blanketed by a blue haze resulting from the eruption of the Laki Volcano in the European republic of Iceland in the summer of 1984
C: major eruptions cause the atmosphere to become cooler than it would be otherwise.
WEAKEN: some eruptions do not cause cooler; the cooler weather is not caused by eruptions
答案:C
(A) The cooling effect triggered by volcanic eruptions in 1985 was counteracted by an unusual warming of Pacific waters.--------------how the cooling was counteracted is not weaken the cooling weather that eruptions cause
(B) There is a strong statistical link between volcanic eruptions and the severity of the rainy season in India.--------------the correlation between eruption and rain is irrelevant. nothing about the cooling
(C) A few months after El Chichn's large eruption in April 1982, air temperatures throughout the region remained higher than expected, given the long-term weather trends.------------CORRECT.suggest another example about the hotter weather caused by eruption. thus undermine the cooling effect of the eruption
(D) The climatic effects of major volcanic eruptions can temporarily mask the general warming trend resulting from an excess of carbon dioxide in the atmosphere.------------how to mask the general trend is not the case, not the issue.
(E) Three months after an early springtime eruption in South America during the late 19th century, sea surface temperatures near the coast began to fall.-------------still support the cooling effect of eruption

3. 23'
P: Many microwave ovens, however, have built-in clocks and so use some electricity even when they are not in use.  The clocks each consume about 45 kilowatt-hours per year
C: households whose microwave oven has no built-in clock use 45 kilowatt-hours per year less, on average, than do comparable households whose microwave oven is otherwise similar but has a built-in clock.
ASSUMPTION: build-in clock is just the only difference between those family
答案:D
(A) Households that do not have a microwave oven use less energy per year, on average, than do households that have a microwave oven.---------------the comparison between households with microwave and households without microwave is irrelevant
(B) Microwave ovens with a built-in clock do not generally cost more to buy than microwave ovens without a built-in clock.------------cost is not discussed. we just focus on electricity usage.
(C) All households that have a microwave oven also have either a gas oven or a conventional electric oven.------------whether they have gas or conventional oven does not matter the more electricity used by built-in clock ovens
(D) Households whose microwave oven does not have a built-in clock are no more likely to have a separate electric clock plugged in than households whose microwave oven has one.--------------CORRECT. if have a separate electric clock, than the separate clock will use electricity. thus it will not be reliable to conclude the 45 kilowatt-hours less electricity.
(E) There are more households that have a microwave oven with a built-in clock than there are households that have a microwave oven without a built-in clock.--------------the different proportion about different oven does not matter

4. 20'
P: Once an airline successfully implements this method, any attempt to recoup the earlier losses by charging high fares on that route for an extended period would only provide competitors with a better opportunity to undercut the airline's fares.
C: this method of eliminating competition cannot be profitable in the long run
WEAKEN: this method can be profitable for some other reasons
答案:B
(A) In some countries it is not illegal for a company to drive away competitors by selling a product below cost.--------------whether legal or illegal, not the consideration
(B) Airline executives generally believe that a company that once underpriced its fares to drive away competitors is very likely to do so again if new competitors emerge.------------CORRECT. means the tendency of reduce price again threaten competitors come in the market.
(C) As part of promotions designed to attract new customers, airlines sometimes reduce their ticket prices to below an economically sustainable level.-----------------just state the reason for attract customer. but noting about the long-term profitable
(D) On deciding to stop serving particular routes, most airlines shift resources to other routes rather than reduce the size of their operations.-------------did weaken the conclusion, but shift to other routes and not reduce the operation do not provide enough evidence to support the long-term profitability of this specific method. And it is still possible for competitors get into the market again after the price is increased
(E) When airlines dramatically reduce their fares on a particular route, the total number of air passengers on that route increases greatly.----------states the advantage of this method. but nothing about the long-term profitability

5. 28'
P: using a new nontoxic plastic film instead of paint.  The film takes just as long to apply as paint does, but many other maintenance tasks can be carried out at the same time.
C: reduce maintenance time
SUPPORT:
答案:B
(A) Unlike paint, the film gives a milky tone to certain colors.-------------colors are not discussed in the question
(B) At the end of its useful life, the film can be removed much more quickly than paint can.----------CORRECT. clearly support the reduced maintenance time
(C) The film can be applied only by technicians who have received special training.---------how the film can be applied does not affect the maintenance time. there is nothing about the maintenance time
(D) The metal exteriors of airplanes have to be protected from high temperatures and caustic chemicals such as exhaust gases.---------how the metal exteriors have to be protected does not affect the maintenance time.
(E) Even at speeds considerably higher than the normal speed of a passenger jet, the film remains securely attached.-------------states the advantages of film, but nothing about the maintenance time.

作者: 艾筱豆    时间: 2013-9-22 21:29
1’17’’
Premise: result plane veered off course
Reason: a passenger put on a laptop
Weaken or least strengthen : other tools or reasons caused the plane veered off   
                       Or the plane and the laptop do not affect each other
B
13 because the eruption  the weather become colder
Weaken:the two are not related
Or there is another reason to affect the weather
A
14 Build in clock oven   oven without build in clock
D
15plan aim to drive off competitor
B
16 film is nontoxic so the maintenance can be done at the same time
B

作者: w.melhere    时间: 2013-11-3 16:27
今天被SC虐了...想来逻辑挽回点颜面...结果...逻辑也做的好烂啊!!

1. 1'53  没有完全理解 'least strengthen'
2. 1‘50’ 错了....题目最后一句话没有理解,应该是:相对于其他可能,LV的排放是变冷的主要原因。题目想要削弱,即找他因。选择C。
3. 1‘14
4. 1’44 又错。觉得E还是很有诱惑,题目本身也确实有点绕。
题目说用降价的办法驱逐竞争者不能长久获利,而在后期赚回loss的时候也会给竞争者使用降价竞争的机会。想要削弱,应该直接针对降价的手段。B选项说,这个手段是可以再一次使用的,不断驱逐新进入的竞争者。而E则说在使用时可能因为吸引了很多消费者而获利。
相比较而言,B更为直接。题目中并没有要求提到获利的问题,E有点发散了。
作者: yuehuasunday    时间: 2013-11-22 16:25
2-2

1. Background: Modern navigation systems, which are found in most
of today’s commercial aircraft, are made with
low-power circuitry, which is more susceptible to
interference than the vacuum-tube circuitry found in
older planes.


Premise: During landing, navigation systems
receive radio signals from the airport to guide the
plane to the runway.


Conclusion: Recently, one plane with
low-power circuitry veered off course during landing,
its dials dimming, when a passenger turned on a
laptop computer. Clearly, modern aircraft navigation
systems are being put at risk by the electronic
devices that passengers carry on board, such as
cassette players and laptop computers.

(E) Correct

2.

Background: An unusually severe winter occurred in Europe

Premise: after the continent was blanketed by a blue haze resulting from the eruption of the Laki Volcano in the European republic of Iceland in the summer of 1984.

Conclusion: Thus, it is evident that major eruptions cause the atmosphere to become cooler than it would be otherwise.

© Correct

3.

Background: Most household appliances use electricity only when in use.  Many microwave ovens, however, have built-in clocks and so use some electricity even when they are not in use.

Premise: The clocks each consume about 45 kilowatt-hours per year.

Conclusion: Therefore, households whose microwave oven has no built-in clock use 45 kilowatt-hours per year less, on average, than do comparable households whose microwave oven is otherwise similar but has a built-in clock.

(D) Correct

4.

Background: Some airlines allegedly reduce fares on certain routes to a level at which they lose money

Premise: n order to drive competitors off those routes.  However, this method of eliminating competition cannot be profitable in the long run.

Conclusion:  Once an airline successfully implements this method, any attempt to recoup the
earlier losses by charging high fares on that route for an extended period would only provide competitors with a better opportunity to undercut the airline's fares.

(B)

5.

Background: When an airplane is taken out of service for maintenance, it is often repainted as well, and during the repainting no other maintenance work can be done on the plane.

Premise:  In order to reduce maintenance time, airline officials are considering using a new nontoxic plastic film instead of paint.

Conclusion: The film takes just as long to apply as paint does, but many other maintenance tasks can be carried out at the same time.

(B)
作者: 苏错    时间: 2013-11-23 21:06
11.23
[精练]
background:low-power circuitury of modern systems is more susceptible than the vacuum-tube in order planes.
permise:a plane veered off because a passenger turned on a computer.
conclusion; modern systems are being put at risk by the new devices

least strengthen= irrelevant or weakn(other possbile reasons for the veering off)

A.strengthen could possible lead to
B.strengthen data
C.irrelevant (相关 证明原因不在了,则结果也不会发生)
D.strengthen
E.correct no effect

[逻辑链]
1.eruption——a blue haze ——severe winter occurred  so, eruption--atmosphere become cooler
weaken 实事证明逻辑链中间断掉 C

2.microwave ovens use electricity when they are not in use.  45 kilowatt
so, has no clock < has a clock
assumption: 无其他影响因素导致结果不同
D

3.method not profitable. recoup the loss  -give the competitors a opportunity to undercut fares.
weaken this result cannot happen or lead to another worse result.
B

4.to reduce maintenance time  use a plastic film.
方法可行:B
作者: tkl    时间: 2013-12-12 15:47
 12
 59’
 Modern is easier  to be influenced than the old ones, proved by an accident as a result of using computer
 
 Background: Modern navigation systems, which are found in mostof today’s commercial aircraft, are made withlow-power circuitry, which is more susceptible tointerference than the vacuum-tube circuitry found inolder planes. During landing, navigation systemsreceive radio signals from the airport to guide theplane to the runway.
 
Premise:Recently, one plane withlow-power circuitry veered off course during landing,its dials dimming, when a passenger turned on alaptop computer.

Conclusion :Clearly, modern aircraft navigationsystems are being put at risk by the electronicdevices that passengers carry on board, such ascassette players and laptop computers.

Prephrase: veer off course when someone used a computer--electronic devices make the modern system risk

Least strengthen: (other factors that can lead to the veer off or unrelated)
A when the supposed cause was eliminate, the problem was solved
That means it is the very supposed cause that lead to the problem.--strengthen
B add information that make sure this suppose cause-effect corse happen (computer interfered navigate system )---strengthen
C remove other related factors that would interfere the course--strengthen
D add information ---strengthen
E_-----无关选项---correct

1345’
An compelling severe winter occurred as a result of volcano eruption ----eruption make the atmosphere cooler than it would be otherwise
Weaken : un usually severe winter did not occurred once th continent was blanketed either.
Background:
Premise : An unusually severe winter occurred in Europe after the continent was blanketed by a blue haze resulting from the eruption of the Laki Volcano in the European republic of Iceland in the summer of 1984. Conclusion :Thus, it is evident that major eruptions cause the atmosphere to become cooler than it would be otherwise.2’
A --unrelated
B it talked about the relation between eruption and severity of rainy season ,while the core in this question is about the relation between eruption and the temperature -unrelated
C another example that eruption cause the higher temperature ---weaken--correct
D unrelated
E it talked about sea surface temperature while in this question we talk about the air temperature--unrelated

14
1’
The microwave with clock will use electricity 45 kilowatt-hours a year when even is not put into use . While microwave without clock use no electricity when not in use.-----the former will use 45 kilowatt-hours a year more on average than the latter
Background:Most household appliances use electricity only when in use. Many microwave ovens, however, have built-in clocks and so use some electricity even when they are not in use.
Premise: The clocks each consume about 45 kilowatt-hours per year.
Conclusion:Therefore, households whose microwave oven has no built-in clock use 45 kilowatt-hours per year less, on average, than do comparable households whose microwave oven is otherwise similar but has a built-in clock.

Prephrase: When not in use , microwave use no electricity , clock use 45 electricity--the microwave with clock will cost 45 more than those without clock on average

D 2’
A the question talk about the housewives who have a microwave with a clock and the housewives who have a microwave without a clock .and it did compare the housewives with a microwave and the housewives without a microwave-----unrelated
B the question compared the quantity of energy cost by the microwave rather the cost of microwave ---unrelated
C it did not talk about the kind of oven ---unrelated
D assumption ---correct
E weaken the conclusion .

15
1’27
Airline want to lower fares at a cost of losing money to eliminate competitors, but they will fail in the long run  , cause when this method succeeds at first ,competitor will enter again during the time they ask higher fares to recoup the earlier lost.
Weaken
Potential answer: the airline is a industry that can exit easily but difficult to enter.

background:Some airlines allegedly reduce fares on certain routes to a level at which they lose money, in order to drive competitors off those routes.
Premise: Once an airline successfully implements this method, any attempt to recoup the earlier losses by charging high fares on that route for an extended period would only provide competitors with a better opportunity to undercut the airline's fares.
Conclusion:However, this method of eliminating competition cannot be profitable in the long run.
B/

A the question talk about the result of reducing fares rather the legality of lowing the fares.-----unrelated
B remove the premise----correct
C the level of reducing fares is not discussed in this argument --unrelated
D whether other airlines would size of their operation is not discussed---unrelated
E this question talk about the relation between the fare reduction and the competitors rather the relation between the fare reduction and the quantity of passengers.


16
37’
The old plane can not be fixed while painting , the film that take painting’s place can make the maintenance available and the cost the same time with painting.so the film may help reduce maintenance time
Strengthen:
The film will not make it harder to fix


Background:When an airplane is taken out of service for maintenance, it is often repainted as well, and during the repainting no other maintenance work can be done on the plane.
Conclusion :In order to reduce maintenance time,
Premise: airline officials are considering using a new nontoxic plastic film instead of paint. The film takes just as long to apply as paint does, but many other maintenance tasks can be carried out at the same time.

B
A the order in this argument is reduce maintenance time, it has nothing to do with color----unrelated
B it compared the time of removing paint and film, indirectly add information that the time will be saved by using film----correct
C it set a higher bars using film----weaken
D metal exteriors are not discussed , it may intend to imply us that the plastic did not require the same condition , but since it did not express it out , it can count as a information  that can be considered---unrelated
E unrelated


作者: cyndichiang    时间: 2014-4-22 14:31
精炼:49‘’
premise:  one plane with low-power circuitry veered off course when a passenger turned on a laptop computer
conclusion: modern aircraft navigation systems are being put at risk by the electronic devices that passengers carry on board
Q: LEAST strengthen
prephrase: 果推因at the same time ,this plane is undergoing other thing that may lead to the sudden change in direction
(A) After the laptop computer was turned off, the
plane regained course and its navigation
instruments and dials returned to normal.----strengthen; this statement identifies that when computer turns on ,the plane would veer off
(B) When in use all electronic devices emit
electromagnetic radiation, which is known to
interfere with circuitry.-----strengthen; this statement shows that the electronic devices do emit radiation
(C) No problems with navigational equipment or
instrument dials have been reported on
flights with no passenger-owned electronic
devices on board.-----strengthen; 无因无果
(D) Significant electromagnetic radiation from
portable electronic devices can travel up to
eight meters, and some passenger seats on
modern aircraft are located within four meters
of the navigation systems.-----this statement shows that eletromagnetic radiation does influence navigation system
(E) Planes were first equipped with low-power
circuitry at about the same time portable
electronic devices became popular.-----correct ; other factors happened at the same time thus weaken this factor's influence分析错了;portable electronic devices 不是另外一个东西吗?不过仔细看了,确实是无关的,因为没有说这种portable electronic devices was applied to this plane.


逻辑链
1.23‘’
premise: the eruption of the Laki Volcano caused the an unusually severe winter
conclusion: major eruptions cause the atmosphere to become cooler
Q:Weaken
prephrase: individual case cannot represent the general condition
may be the case in 1984 is a special one

2.39''
premise:built-in clocks will each consume 45 kilowatt/year when they are not in use
conclusion: on average,microwave oven with no built-in clock use 45 kilowatt/year less than do comparable microwave with built-in clock
Q: Assumption
prephrase: 没有gap,很可能是削弱取非
people will not more frequently use microwave oven with no built-in clock than with built-in

3. 47‘’
premise: airplane firm reduce the price in order to drive competitors off
premise:recouping the earlier losses by charging high fares would lead competitors to undercut the fares(side-effect)
conclusion: this method cannot be profitable in a long run
Q: weaken
prephrase:  证明方案可行competitors will not undercut the fare?
                side-effect is not existed since they will not charge high fares in a long term

4. 19''
premise: when a new nontoxic plastic film is applied when an airplane is taken out of service for maintenance, many other maintenance tasks can be carried out at the same time.
conclusion:it can help reduce maintenance time
Q: support
prephrase: other things about nontoxic plastic will not cost more time

作者: gmatkenny    时间: 2014-5-23 10:45
UlysessHope 发表于 2011-12-10 10:49
逻辑链答案:CDBB 精练解析:The conclusion of the argument is based on the causal assumption thatelec ...

                         不错!
作者: jolenewjy    时间: 2014-5-30 04:25
33’’
B: airplane need this device to guide airplane landing
P: recently, there is one accident where the plane is off course due to one passenger using laptop
C: navigation system is at higher risk due to the use of electronic devices.  
Predict: there are other reason why it accident, passenger will not be allowed to use the devices.
Choice:E
13.  16’’
P: eruption caused haze in the summer
C: eruption will make atmosphere colder than it would be.
Prediction: there is special environment changes happened along with or before that.
Choice: C
14. 22’’
P: microwave have built in clock still use electricity when not in use
C: microwave does not have built in clock use less power than ones that have built clock.
Prediction: the power both type of microwave use when in use is the same.
Choice: D
15. 28’’
B: airlines reduced ticket price to beat competitors to a level they loss money
P: Once airline try to recope money from other routes, competitor airlines could offer lower price.
C: airlines will not be profitably in the long run adopting this price reduction method.
Prediction: there will be loyal customer and other ways to recope money not soly on the ticket.
Choice: B
16. 26’’
B: when airplane is taken out of service, airplane need to be repaint and no other work can be done at the same time
P: The file stays as long as paint does and other task can be at the same time.
C: officials should use this film instead of paint.
Prediction: Other benefit of the film than paint.
Choice: B

作者: zixuan1017    时间: 2017-6-4 15:03
感谢分享!               
作者: May97    时间: 2018-3-12 07:37
EADEB
作者: 云栈    时间: 2019-11-13 05:54
1.        43s
B: 当今大多数商用飞机中都使用的现代导航系统是用低功率电路制成的,该电路比老式飞机中的真空管电路更容易受到干扰。
P: 在着陆期间,导航系统会从机场接收无线电信号,以将飞机引导至跑道。
P: 最近,一架带有低功率电路的飞机在降落过程中转向偏离航线,当乘客打开笔记本电脑时,其表盘变暗。
C: 显然,现代飞机导航系统正受到乘客携带的电子设备(例如卡带播放器和膝上型计算机)的威胁。
推测:证明navigation受到游客的电子产品的影响不大;或者与原题无关;
选项分析:E
A: 支持,说明laptop就是干扰的原因
B:支持,说明电子设备的电磁感染circuitry
C: 支持,说明没有电子设备就不会出问题
D: 支持,说明电磁干扰的范围大于和navigation的距离
E:无关

2.        37s
P: 1984年夏季,欧洲大陆冰岛被拉克火山喷发而造成的蓝色薄雾笼罩,欧洲发生了异常异常严峻的冬天。
C:因此,明显的是,大爆发使大气变得比以前更凉。
推测:举例说明有的eruption没有导致大气更冷;说明寒冬是因为别的原因导致的
选项分析:C

3.        48s
P: 大多数家用电器仅在使用时才用电。但是,许多微波炉都有内置时钟,因此即使不使用它们也会消耗一些电量。
P: 每个时钟每年消耗约45千瓦时。
C:因此,没有内置时钟的微波炉的家庭平均每年要比没有微波但具有内置时钟的可比家庭少使用45千瓦时。
推测:微波炉上没有时间不会导致人用更多的微波炉;微波炉上有没有时间不会影响人买另一个闹钟
选项分析:D

4.        50s
P: 据称,一些航空公司将某些航线的票价降低到他们亏损的水平,以驱使竞争对手离开那些航线。
C:但是,从长远来看,这种消除竞争的方法无法盈利。
P: 航空公司成功实施此方法后,任何试图通过在较长时间内收取高额票价来弥补先前损失的尝试,只会为竞争对手提供降低航空公司票价的更好机会。
推测:降低票价也会吸引乘客购买该公司其他的正常票价的航班从而带来收益,航空公司不需要调回票价
选项分析:B

5.        34s
P: 当使飞机停止维修时,通常也会对其进行重新粉刷,并且在重新粉刷期间,飞机上无法进行其他维护工作。
P: 为了减少维护时间,航空公司官员正在考虑使用新的无毒塑料薄膜代替油漆。
P: 涂膜的时间与涂料的涂膜时间一样长,但是可以同时执行许多其他维护任务。
推测:使用plastic film维修的时间不会更长
选项分析:B

作者: Lincy123    时间: 2020-6-3 08:48
P:近期一家lpc飞机当乘客打开电脑时veered off
C:现代飞机导航系统因为电子设备而有风险
Least strengthen:无关或Weaken:找他因,或证明电子设备无风险
A) 当乘客关掉电脑时回复正常。Strengthen
B) 当使用电子设备时会发出电磁辐射,影响c。strengthen
C) 没有电子设备时导航设备没有问题。Strengthen
D)电磁辐射可以影响8米,一些乘客坐在离设备4米的地方。 有点不足
E) 在飞机安装lpc的同时,便携式电子设备流行起来,无关。Correct

P:火山爆发后的bh导致欧洲冬天很冷
C:火山爆发会让大气层变得更冷
A) 火山爆发的cooling effect被太平洋暖流抵消了。待定
B) 火山爆发和印度雨季关联性强,无关
C) 火山爆发几个月后,空气温度比预期中高,导致长期气候趋势。Correct
D)火山爆发的气候影响会笼罩由于二氧化碳过多导致的变暖趋势。
E) 在火山爆发三个月后,海水表面温度下降

P:有钟的微波炉每天会消耗45千瓦
C:有没有钟的微波炉的家庭比有钟的省电45千瓦
Assumption:没有钟的微波炉没有其他用电的地方
A) 没有微波炉的家庭用电量更少。无关
B) 有钟的微波炉更便宜。无关
C) 有微波炉的家庭通常都有煤气炉,无关
D)没有钟的微波炉的家庭更可不能有一个单独的钟。Correct
E) 更多家庭有有钟的微波炉。无关

P:一旦减少了就很难恢复到原来价格,恢复价格会给竞争对手降价机会
C:亏本减少特定航线费用的方法不能长期获利
Weaken
A) 在一些国家亏本卖东西是合法的。无关
B) 一旦公司采取降价措施,未来有新竞争者的时候很可能再这么做。无关(x)correct
博弈论:竞争对手知道如果进入市场,对方同样会用降价措施把自己挤出市场,则竞争者不会进入市场,因此可以长期获利
C) 为了吸引新顾客,航空公司有时低价买票。无关
D)为了停止特定航线,航空公司把资源移到其他航线而不是减少他们的运作规模。
E) 当航空公司减少特定航线费用时,这条线的乘客会大幅度增长。Correct(x)
销量上升并不代表能长期获利

P:用塑料膜替代油漆,在换膜的同时可以做其他事情
C:减少维修时间
Strengthen
A) 膜有特定颜色。无关
B) 膜可以比油漆更快去除。Correct
C) 膜只能经过专门训练的人安装,无关
D)飞机外壳需要避免高温。无关
E) 即使速度更高,膜都维持安全。无关






欢迎光临 ChaseDream (https://forum.chasedream.com/) Powered by Discuz! X3.3